# Prime Producing Polynomials and Unique Factorization (Lec. 11, Tuesday, February 23) :::{.remark} Today: chapters 11 and 12. ::: ## Ch. 11: Prime Producing Polynomials and Unique Factorization :::{.remark} 18th century observation by Euler about the following polynomial: \[ f(x) \da x^2 -x + 41 .\] Goldbach proved that it's impossible for any polynomial $g\in \ZZ[x]$ to have *every* output prime. Euler noted that this $f$ produces quite a few: for $x=1, \cdots, 40$, the output $f(x)$ is prime, but $f(41) = 41^2 - 41 + 41 = 41^2$ is not. Let's define a variant: for $q$ a positive integer, set \[ f_q(x) \da x^2 - x + q .\] Note that $f_q(q) = q^2$, so eventually the output is composite. We'll say $f_q$ is **optimal** if $f_q(x)$ is prime for all integers $0 < x < q$. As an example, $q=41$ was optimal. ::: :::{.theorem title="Rabinowitz"} Let $q \geq 2 \in \ZZ^{> 0}$ and let $d = \Delta(f_q) = 1-4q$ be the discriminant of $f_q$. Assume that $d$ is squarefree, then $f_q$ is optimal if and only if $\zadjoin{ {1 + \sqrt{d}\over 2 }}$ is a UFD. [^actual_ring_of_ints] [^actual_ring_of_ints]: Note that this is equal to $\ZZ_K$ when $K\da \QQ( \sqrt{d} )$. ::: :::{.example title="of a ring of integers that is a UFD"} For $q=41, d = -163$ and thus $\zadjoin{ {1 + \sqrt{-163} \over 2}}$ is a UFD. ::: :::{.proof title="$\impliedby$"} > Big idea: uses that $\min_\tau(x) = f_q(x)$ and remembering that how $\min_\tau(x) \mod p$ factors is exactly how \( \gens{ p } \) factors into prime ideals. Assume \( \zadjoin{ \tau } \) is a UFD, where \( \tau\da {1 + \sqrt{d} \over 2 } \). Toward a contradiction, suppose $f_q(x)$ is composite for some $01$, so $p \geq q- 1/4$. Since $p, q\in \ZZ$ we can strengthen this to $p \geq q$. But $p$ was the *least* prime factor of $f_q(x) < q^2$ which was composite, so this is a contradiction. $\contradiction$ ::: :::{.remark} The forward direction is harder here. ::: :::{.proof title="$\implies$"} We'll prove something stronger. Assume $f_q(x)$ is prime whenever \[ 1 \leq x \leq {1\over 2} \sqrt{ \abs{d} \over 3 } + {1 \over 2} ,\] then we'll prove that $\ZZ_K$ is a PID and hence a UFD. Note that this is stronger because the range is smaller than $0 \sqrt{\abs{d} \over 3 } \geq p .\] This contradicts $f_q(x)$ being prime. $\contradiction$ ::: So assuming $f_q(x)$ is prime for $1 \leq x \leq {1 \over 2 } \sqrt{\abs{d}\over 3 } + {1 \over 2}$, we showed that every "small" prime up to \( \sqrt{\abs{d}\over 3 } \) is inert. Suppose $P$ is a prime ideal above $p$, then since $p$ is inert, $P = \gens{ p }$ is generated by a prime. But we'll just use a slightly weaker conclusion: $P$ is principal. :::{.theorem title="When the class group is generated by small primes"} Let $d$ be a negative squarefree integer with $d \equiv 1 \mod 4$ (such as the $d$ we are looking at). Then $\Cl(\ZZ_K)$ is generated as a group by $[P]$ where $P$ runs over all prime ideals above primes $p \leq \sqrt{\abs{d}\over 3 }$. ::: Given this theorem, we are done: in our situation, all such $[P]$ are trivial in the class group since they are principal, which makes $\Cl(\ZZ_K) = 1$ and every ideal is principal. ::: ## Proof of Rabinowitz's Theorem :::{.remark} It just remains to prove the above theorem. We'll use the following: ::: :::{.proposition title="Almost Euclidean Domains"} Take the same assumptions on $d$ as above. Then for each $\theta \in K = \QQ( \sqrt{d} )$, there is a positive integer $t \leq \sqrt{\abs{d}\over 3 }$ and a $\xi \in \ZZ_K$ with norm \( N(t\theta - \xi) < 1 \). ::: :::{.remark} This is slightly technical. In words: for any element in your quadratic field, you can approximate it by an *integer* of your field, possibly after a small $t$ dilation. Note that we saw a similar condition for the Euclidean algorithm, namely that $t=1$ always sufficed. ::: :::{.proof title="of proposition"} Write \( \theta = a + b \tau \) where we don't necessarily know $\theta \in \ZZ_K$ (although this would make the statement trivial), but $a, b \in \QQ$. We want to find an appropriate $t$ where $\xi \da A+ B \tau$ for $A, B \in \ZZ$. Multiplying the inequality out using the definition of the norm results in \[ \qty{ (ta - A) + \qty{tb - B \over 2} }^2 + \abs{d} \qty{ tb - B \over 2 }^2<1 .\] We'll start by making the second term small by making $tb$ close to an integer (where $b$ is fixed) and choosing $B$ to be that closest integer. We can choose $t \leq \sqrt{\abs{d}\over 3 }$ with $\norm{tb} < 1 / \sqrt{\abs{d}\over 3 }$, where the norm is the distance to the nearest integer. Why can we do this? This is Dirichlet's approximation theorem, where we could choose $t\leq N$ such that this norm was bounded by $1/(N+1)$, and we can take $N \da \floor{ \sqrt{\abs{d}\over 3 } }$. How do we choose $A, B$? Choose $B$ such that $\norm{tb}$ satisfies the above inequality to obtain \[ B\in \ZZ,\quad \abs{tb - B}< 1 / \sqrt{\abs{d}/3 } .\] Then considering the second term in the original equation, we get \[ \abs{d} \qty{ tb -B \over 2 }^2 < \abs{d}\qty{1\over 4}\qty{1 \over \abs{d}/3} = {3\over 4} ,\] so it suffices now to choose $A$ such that the first term is bounded by $1/4$. Why can we do this? We have control over $A$, so we can simply choose it freely to shift the inner quantity into the interval $[-1/2, 1/2]$, i.e. \[ \abs { ta - A + \qty{tb- B \over 2} } \leq {1\over 2} ,\] and then squaring yields the desired bound. ::: :::{.proof title="of theorem"} We have $d \equiv 1 \mod 4$ and we want to show that the class group is generated by small primes $p\leq D\da \sqrt{ \abs{d} / 3 }$. Let $I$ be a nonzero ideal of $\ZZ_K$, we'll find a nonzero ideal $J$ such that $[I] = [J]$ and $J$ is a product of primes above $p$ (which have the appropriate upper bound). In this case, $[J]$ and thus $[I]$ will factor as the product of those primes, and is thus in the subgroup generated by small primes. Fix \( \beta\in I \) nonzero of minimal norm. :::{.claim} For any \( \alpha \in I \) there is a $t\leq D$ with \( \gens{ t \alpha } \in \gens{ \beta } \). ::: :::{.remark} How to think about this: when the field is Euclidean with respect to the norm, it is a PID. How do you find generators? By taking a nonzero element \( \beta \) of minimal norm in the ideal. Then any element of $I$ would be in \( \beta \). Here we have an almost-Euclidean property, where elements of $I$ can be hit with a small dilation to land in this principal ideal. ::: :::{.proof title="of claim"} So apply the last element to the element \( \alpha/ \beta \) to pick $t\leq D$ and $\xi \in \ZZ_K$ with \( N( t \qty{\alpha\over \beta} < 1 \). Multiplying through by $N( \beta)$ yields \[ N ( t \alpha - \beta\xi ) < N( \beta ) .\] Note that the inner term on the left-hand side is in $I$, since \( \alpha, \beta\in I \). This is an element of $I$ of norm less than the norm $\beta$, but by minimality this can only happen if $t \alpha - \beta\xi =0$ and thus \( t \alpha = \beta\xi \in \gens{ \beta } \). ::: Now let \( T \da \floor{D} !\), where we take the factorial. Then for any \( \alpha\in I \) we have \( T \alpha \in \gens{ \beta } \). Why? We already know *some* factor of $T$ is a multiple of \( \beta \), and multiplying by other factors doesn't take it out of the ideal. Since this was true for every \( \alpha\in I \) we have \( TI \subseteq \gens{ \beta } = \beta\ZZ_K \). Define $J \da \qty{T/ \beta} I$, where noting that $T\in \ZZ^{> 0}$, this is just a dilation of $I$. Then $J \subseteq \ZZ_K$, since \[ TI \subseteq \beta\ZZ_K \overset{\cdot \beta\inv}{\implies } \beta\inv T I \subseteq \beta\inv \beta \ZZ_K = \ZZ_K .\] Moreover, $J \normal \ZZ_K$ is an ideal, since it's a dilation of an ideal, $[J] = [I]$ since they're related by dilation, and $J$ contains $\qty{T / \beta } \beta = T$. Since to contain is to divide, we have $J \divides \gens{ T }$. Recall that $T$ was a product of integers, so however \( \gens{ T } \) factors into prime ideals, every such prime ideal will lie above an actual prime no bigger than $D$. You can factor \( \gens{ T } \) by first factoring $T$ into prime integers, then break them up into prime ideals, all of which would have norm bounded by $D$. ::: :::{.remark} ::: :::{.remark} This proves Rabinowitz's theorem. \ This says that being an optimal prime is entirely equivalent to a certain ring being a UFD. Are there optimal examples other than $q=41$? It turns out that there are *no* optimal $f_q$ for $q>41$, which is not easy to prove. This didn't happen until the 20th century, by folks interested in the UFD side of this statement: ::: :::{.theorem title="Baker-Heegner-Stark"} $\ZZ_K$ is not a UFD if $K \da \QQ( \sqrt{d} )$ with $d$ squarefree with $d<-163$. ::: :::{.remark} So remarkably, there are *not* infinitely many examples for which the ring of integers is a UFD. Thus the class number only takes on the value 1 for finitely many fields. What about for 2? This also only happens finitely often. In fact, for *any* fixed $h$, there are only finitely many imaginary quadratic fields with class number $h$. This follows from the fact that $\Cl(\QQ( \sqrt{d} )) \approx d^{1/2}$, which increases as $d$ does. It's still hard to determine for a given $h$ which values of $d$ appear, partially because the last statement is *ineffective*[^updated_effective] in the sense that there aren't constants to put into the asymptotic statement. [^updated_effective]: Note that this may not be true as of 2020! See Griffin, M., & Ono, K. (2020). Elliptic curves and lower bounds for class numbers. Journal of Number Theory, 214, 1-12. ::: :::{.remark} What about real quadratic fields? An expert on this will be joining us here at UGA starting Fall 2021. The situation is expected to be very different, and a conjecture is the following: ::: :::{.conjecture} The real quadratic field $\QQ(\sqrt{d})$ is a UFD most of the time. ::: ## Lattice Points :::{.remark} This corresponds to chapter 12. Everything we've done up until now has been for quadratic fields. After this chapter, we'll start anew and rebuild everything for general number fields. ::: :::{.definition title="Lattice Point"} A **lattice point** in $\RR^n$ is a point in $\ZZ^n$. ::: :::{.question} Given a region $R \subseteq \RR^n$, how many lattice points does it contain? I.e., how large is the sum \[ \sum_{\vector v \in \ZZ^n} \chi_R(\vector v) .\] ::: :::{.remark} A first guess might be that this is approximately $\vol(R)$. To see why, one can try just choosing to count any squares for which the lower-left point is contained in $R$ and adding up the areas: \pgfmathsetseed{12} \begin{tikzpicture} \draw[step=1.0,black,thin] (0.5,0.5) grid (9,7); \draw [shade, top color=blue, bottom color=white, fill opacity=.1, decoration={random steps,segment length=2cm,amplitude=.75cm}, decorate, rounded corners=.3cm] (2, 3) -- (4,6) -- (6,6) -- (7.6,1.5) -- (4, 1) -- (2, 3); \node[fill=black, circle, inner sep=0.1cm] at (4,2) {}; \node[fill=black, circle, inner sep=0.1cm] at (5,2) {}; \node[fill=black, circle, inner sep=0.1cm] at (6,2) {}; \node[fill=black, circle, inner sep=0.1cm] at (7,2) {}; \node[fill=black, circle, inner sep=0.1cm] at (3,3) {}; \node[fill=black, circle, inner sep=0.1cm] at (4,3) {}; \node[fill=black, circle, inner sep=0.1cm] at (5,3) {}; \node[fill=black, circle, inner sep=0.1cm] at (6,3) {}; \node[fill=black, circle, inner sep=0.1cm] at (7,3) {}; \node[fill=black, circle, inner sep=0.1cm] at (3,4) {}; \node[fill=black, circle, inner sep=0.1cm] at (4,4) {}; \node[fill=black, circle, inner sep=0.1cm] at (5,4) {}; \node[fill=black, circle, inner sep=0.1cm] at (6,4) {}; \node[fill=black, circle, inner sep=0.1cm] at (7,4) {}; \node[fill=black, circle, inner sep=0.1cm] at (4,5) {}; \node[fill=black, circle, inner sep=0.1cm] at (5,5) {}; \node[fill=black, circle, inner sep=0.1cm] at (6,5) {}; \draw[draw=black, fill=green, fill opacity=0.1] (4, 2) rectangle ++(1,1); \draw[draw=black, fill=green, fill opacity=0.1] (5, 2) rectangle ++(1,1); \draw[draw=black, fill=green, fill opacity=0.1] (6, 2) rectangle ++(1,1); \draw[draw=black, fill=green, fill opacity=0.1] (7, 2) rectangle ++(1,1); \draw[draw=black, fill=green, fill opacity=0.1] (3, 3) rectangle ++(1,1); \draw[draw=black, fill=green, fill opacity=0.1] (4, 3) rectangle ++(1,1); \draw[draw=black, fill=green, fill opacity=0.1] (5, 3) rectangle ++(1,1); \draw[draw=black, fill=green, fill opacity=0.1] (6, 3) rectangle ++(1,1); \draw[draw=black, fill=green, fill opacity=0.1] (7, 3) rectangle ++(1,1); \draw[draw=black, fill=green, fill opacity=0.1] (3, 4) rectangle ++(1,1); \draw[draw=black, fill=green, fill opacity=0.1] (4, 4) rectangle ++(1,1); \draw[draw=black, fill=green, fill opacity=0.1] (5, 4) rectangle ++(1,1); \draw[draw=black, fill=green, fill opacity=0.1] (6, 4) rectangle ++(1,1); \draw[draw=black, fill=green, fill opacity=0.1] (7, 4) rectangle ++(1,1); \draw[draw=black, fill=green, fill opacity=0.1] (4, 5) rectangle ++(1,1); \draw[draw=black, fill=green, fill opacity=0.1] (5, 5) rectangle ++(1,1); \draw[draw=black, fill=green, fill opacity=0.1] (6, 5) rectangle ++(1,1); \node at (2.25, 2.25) {$R$}; \node at (8.15, 5.45) {$\approx\mathrm{vol}(R)$}; \node at (2.25, 6.25) {$\ZZ^n \subseteq \RR^n$}; \end{tikzpicture} This isn't exactly right, but would become closer as $R$ grew larger, and the correction term comes from edge effects. For $R \subseteq \RR^n$ and $t\in \RR$, define the dilation \[ tR \da \ts{ t\vector x \st \vector x \in R } .\] ::: :::{.theorem title="The number of lattice points in a region is asymptotically the volume"} Let $R$ be a region in $\RR^n$ which is *Riemann measurable*.[^riemann_measurable] Then the number of lattice points satisfies \[ {1\over t^n} \sum_{\vector v \in \ZZ^n} \chi_{tR} (\vector v) \converges{t\to \infty }\to \vol(R) .\] [^riemann_measurable]: This means that $\chi_R$ should be Riemann integrable, i.e. the bounded region is contained in a rectangle, and integrals over such rectangles converges to what we'll call the volume. ::: :::{.proof title="of theorem"} Notice that the left-hand side can be written as \[ {1\over t^n} \sum_{\vector v \in \ZZ^n} \chi_{tR} (\vector v) = {1\over t^n} = \sum_{\vector w \in t\inv \ZZ^n} \chi_R(\vector w) .\] This has the effect of making the squares partitioning $\RR^n$ finer, the right-hand side is literally the Riemann sum for \[ \int \chi_R(\vector w) \,d \vector w \da \vol(R) .\] ::: :::{.remark} Note that there is a small technicality since $t$ can take on non-integer values, but the limiting behavior is the same. Next time: we've seen that the number of lattice points is sometimes well-approximated by volume, but it's possible to have regions of unbounded volume with no lattice points, e.g. by taking a large ball and deleting all lattice points. It would be nice to have a theorem which guarantee when a region will have lattice points, and Minkowski's theorem will be one such theorem we'll look at next time. :::